0
$\begingroup$

Consider an Amsler surface—that is, a pseudospherical surface containing two straight lines that intersect at a point on the surface. It is known that such a surface is uniquely determined by the two lines; in particular, varying the angle between them yields a one-parameter family of Amsler surfaces.

I computed one numerically and plotted it as a mesh (see the figure). As I vary the initial angle, I observe that as the angle tends to $0$, the surface approaches a surface that looks like a pseudospherical surface of revolution (in fact the pseudosphere itslef).

My question: is there a standard explanation for this phenomenon? I would appreciate any clarification or references (especially since there is a lot of classical results on the subject in different languages).

I increased the resolution in the last case a bit for a better visualization.

$\endgroup$
6
  • $\begingroup$ How did you "compute one numerically"? The "limiting equation" to be solved for the surface (e.g. Theorems 5.1.1 and 5.1.2 in link.springer.com/chapter/10.1007/3-540-44452-1_5 ) are singular (the solution to the associated sine-Gordon equation is trivial). So it is possible that what you see in the limit is dependent on exactly how the limit is taken. It will be helpful if you can specified the equations being solved to generate the mesh. $\endgroup$ Commented Oct 20 at 18:53
  • $\begingroup$ By "pseudospherical surface of revolution" do you mean "pseudosphere" ? That is, the surface of constant Gaussian curvature -1 obtained by rotating a tractrix about the x-axis? $\endgroup$ Commented Oct 20 at 21:58
  • $\begingroup$ @DanielAsimov yes, visually it looks like an actual pseudosphere. $\endgroup$ Commented Oct 21 at 6:55
  • $\begingroup$ @WillieWong I built the Amsler surface using discrete methods and through Lelieuvre vector fields. I add the details soon as an edit. $\endgroup$ Commented Oct 21 at 6:57
  • $\begingroup$ Also, can you rerun your simulation to highlight the two distinguished lines? Right now you have two families, one blue and one red. For the first three images it is pretty easy to see which ones are supposed to be the embeddied lines. But for the "pseudosphere" it is really hard to tell. Note that the pseudosphere does not have any embedded lines, so I am curious where the two lines sit within your final picture. $\endgroup$ Commented Oct 21 at 13:14

1 Answer 1

1
$\begingroup$

Possible answer: $ \DeclareMathOperator\sech{sech} $

The standard tractroid is parametrized as $$(t,\theta) \mapsto (t - \tanh t, \sech t \cdot e^{i\theta}) \in \mathbb{R}\times \mathbb{C} \cong \mathbb{R}^3 $$ One can compute that the unit normal is $$ \vec{n} = (\sech t, \tanh t \cdot e^{i\theta}) $$ and that the two families of asymptote curves are given by $$ \gamma_{\pm,\theta_0}(t) = (t - \tanh t, \sech t \cdot e^{i(\theta_0 \pm t)} ) $$ here the parametrization has unit speed. Observe also that for $t$ very negative, $$ |\gamma_{\pm,\theta_0}(t) - (t + 1,0)| = |(1 + \tanh t,\sech(t))| = 2|1+\tanh t| $$ which decays exponentially as $t\to -\infty$.

In particular, given $t$, choose $\theta_\pm$ such that $e^{i(\theta_\pm \pm t)}$ are equal, the inner product between the two families are $$ \langle \dot{\gamma}_{+,\theta_+}(t), \dot{\gamma}_{-,\theta_-}(t) \rangle = 1 - 2 \sech^2(t)$$ Note that the corresponding angle is order $2 \sech(t)$ which decays exponentially as $t \to -\infty$.

Thus: suppose you are interested in generating the Amsler surface starting with a pair of line segments with lengths $M$ and opening angle $\theta$. Fixing $M$ and taking $\theta\to 0$ you will find that the sliver of the tractroid bounded by $\gamma_{\pm,\theta_\pm}(t)$, where $t\in [t_0,t_0+M]$ with $t_0$ being the negative solution to $1 - 2\sech^2(t_0) = \cos\theta$, is an approximate solution to your problem with error of order $\theta$.

Your generated picture indicates that you are using fixed grid sizes in your mesh, which are much larger than the small initial angle. If this is the case, probably your numerical solution is not that reliable in the case of the small angles. You probably want to adjust your mesh size to be much smaller than the opening angle, so that you can reliably resolve the difference between the two solutions. (Also, looking at the final picture, I don't see obvious candidates for the two "straight lines" that should appear for the Amsler surface; so perhaps you are not prescribing that as boundary condition but instead solving for it dynamically?)

$\endgroup$

You must log in to answer this question.

Start asking to get answers

Find the answer to your question by asking.

Ask question

Explore related questions

See similar questions with these tags.